lawyering iii

Pataasin ang iyong marka sa homework at exams ngayon gamit ang Quizwiz!

Lawyer Logan, while working at the XYZ Law Firm, formerly represented a local bank in connection with a data breach involving customer social security numbers from two years ago, and the representation of the bank is over. Lawyer Logan has since left the XYZ Law Firm, and he now works at the ABC Law Firm. A customer of the bank wants to hire Attorney Ally, who also works at the same ABC Law Firm, to sue the bank for the same data breach. Which of the following is correct?

Both Attorney Ally may represent the customer if Lawyer Logan is screened from the new matter, given no fee from the new matter, and written notice is given to the bank; and Attorney Ally may represent the customer if the bank gives informed consent, confirmed in writing

Paralegal Pat works at the ABC Law Firm, and Paralegal Pat engages in some fraud in connection with a discovery matter in a civil case by intentionally altering key information in a document produced in discovery. Which of the following lawyers in the ABC Law Firm can be responsible for Pat's conduct?

Both Partner A, who is Pat's direct supervisor and who did not make reasonable efforts to review Pat's work in connection with the discovery in this case; and Partner B, a senior lawyer and partner at the firm who does not supervise Pat and who did not work on the case, but who learned of Pat's conduct at a time when its consequences could be mitigated and did not take remedial actions.

Archie and Brandy are involved in a car accident when their two cars collided at an intersection. Archie hires Attorney Amber to represent him in connection with the accident, but Brandy has not hired an attorney and does not plan to do so. After being hired to represent Archie, Attorney Amber calls Brandy on the phone. Out of the following statements, which statement best complies with Attorney Amber's ethical duties?

"I represent Archie in connection with your recent car accident, and I would like to speak to you about what happened. Is an attorney representing you?"

Lawyer Lisa is representing defendant Chris in a criminal matter, where Chris is accused of an armed robbery. Which of the following statements is least problematic for Lawyer Lisa to make to the media about this case?

"There have been negative press reports that my client confessed to the police on the night of the crime that he committed the robbery, but those reports are inaccurate. The police brutally coerced a fake confession from my client."

Which of the following statements would be improper for a defense lawyer to make to the media in connection with an ongoing case where the lawyer is representing a criminal defendant?

"Witness William will testify tomorrow that Defendant Dave is innocent because William was playing cards with Dave at the time of the burglary."

During discovery in a large, commercial, breach of contract lawsuit, the lawyers for both sides have been exchanging hundreds of documents. Lawyer Lucy, who is representing the plaintiff, received a document from the opposing lawyer as part of the discovery in the case, and the document appears to be a confidential defense memo discussing trial strategy for the case. Which of the following is correct?

Lawyer Lucy must promptly notify the opposing lawyer about the memo.

Which of the following checks must be deposited in a law firm's bank account, as opposed to a trust account for clients?

A $15,000 flat fee for a commercial dispute.

Which of the following checks must be deposited in a law firm's operating account, as opposed to a trust account for clients?

A $5,000 flat fee for a criminal matter.

Rule 3.7: Lawyer as Witness

Advocate (a) A lawyer shall not act as advocate at a trial in which the lawyer is likely to be a necessary witness unless: (1) the testimony relates to an uncontested issue; (2) the testimony relates to the nature and value of legal services rendered in the case; or (3) disqualification of the lawyer would work substantial hardship on the client. (b) A lawyer may act as advocate in a trial in which another lawyer in the lawyer's firm is likely to be called as a witness unless precluded from doing so by Rule 1.7 or Rule 1.9.

Rule 4.4: Respect for Rights of Third Persons

Transactions With Persons Other Than Clients (a) In representing a client, a lawyer shall not use means that have no substantial purpose other than to embarrass, delay, or burden a third person, or use methods of obtaining evidence that violate the legal rights of such a person. (b) A lawyer who receives a document or electronically stored information relating to the representation of the lawyer's client and knows or reasonably should know that the document or electronically stored information was inadvertently sent shall promptly notify the sender.

Rule 3.3: Candor Toward the Tribunal

(a) A lawyer shall not knowingly: (1) make a false statement of fact or law to a tribunal or fail to correct a false statement of material fact or law previously made to the tribunal by the lawyer; (2) fail to disclose to the tribunal legal authority in the controlling jurisdiction known to the lawyer to be directly adverse to the position of the client and not disclosed by opposing counsel; or (3) offer evidence that the lawyer knows to be false. If a lawyer, the lawyer's client, or a witness called by the lawyer, has offered material evidence and the lawyer comes to know of its falsity, the lawyer shall take reasonable remedial measures, including, if necessary, disclosure to the tribunal. A lawyer may refuse to offer evidence, other than the testimony of a defendant in a criminal matter, that the lawyer reasonably believes is false. (b) A lawyer who represents a client in an adjudicative proceeding and who knows that a person intends to engage, is engaging or has engaged in criminal or fraudulent conduct related to the proceeding shall take reasonable remedial measures, including, if necessary, disclosure to the tribunal. (c) The duties stated in paragraphs (a) and (b) continue to the conclusion of the proceeding, and apply even if compliance requires disclosure of information otherwise protected by Rule 1.6. (d) In an ex parte proceeding, a lawyer shall inform the tribunal of all material facts known to the lawyer that will enable the tribunal to make an informed decision, whether or not the facts are adverse.

Assuming that a trial is still ongoing, which of the following things must the plaintiff's attorney disclose in a civil lawsuit?

A case from the controlling jurisdiction known to the plaintiff's attorney to be directly adverse to the plaintiff's case and not disclosed by the defendant's attorney; All materials facts, if the proceeding is ex parte.

Lawyer Logan is defending client Cole in connection with a criminal matter. Which of the following fee agreements would be appropriate for this representation?

A verbal flat fee agreement, whereby Lawyer Logan agrees to handle the entire representation for a flat fee of $20,000.

Rule 3.6: Trial Publicity

Advocate (a) A lawyer who is participating or has participated in the investigation or litigation of a matter shall not make an extrajudicial statement that the lawyer knows or reasonably should know will be disseminated by means of public communication and will have a substantial likelihood of materially prejudicing an adjudicative proceeding in the matter. (b) Notwithstanding paragraph (a), a lawyer may state: (1) the claim, offense or defense involved and, except when prohibited by law, the identity of the persons involved; (2) information contained in a public record; (3) that an investigation of a matter is in progress; (4) the scheduling or result of any step in litigation; (5) a request for assistance in obtaining evidence and information necessary thereto; (6) a warning of danger concerning the behavior of a person involved, when there is reason to believe that there exists the likelihood of substantial harm to an individual or to the public interest; and (7) in a criminal case, in addition to subparagraphs (1) through (6): (i) the identity, residence, occupation and family status of the accused; (ii) if the accused has not been apprehended, information necessary to aid in apprehension of that person; (iii) the fact, time and place of arrest; and (iv) the identity of investigating and arresting officers or agencies and the length of the investigation. (c) Notwithstanding paragraph (a), a lawyer may make a statement that a reasonable lawyer would believe is required to protect a client from the substantial undue prejudicial effect of recent publicity not initiated by the lawyer or the lawyer's client. A statement made pursuant to this paragraph shall be limited to such information as is necessary to mitigate the recent adverse publicity. (d) No lawyer associated in a firm or government agency with a lawyer subject to paragraph (a) shall make a statement prohibited by paragraph (a).

Rule 3.9: Advocate in Nonadjudicative Proceedings

Advocate A lawyer representing a client before a legislative body or administrative agency in a nonadjudicative proceeding shall disclose that the appearance is in a representative capacity and shall conform to the provisions of Rules 3.3(a) through (c), 3.4(a) through (c), and 3.5.

Rule 3.2: Expediting Litigation

Advocate A lawyer shall make reasonable efforts to expedite litigation consistent with the interests of the client.

Rule 3.1: Meritorious Claims & Contentions

Advocate A lawyer shall not bring or defend a proceeding, or assert or controvert an issue therein, unless there is a basis in law and fact for doing so that is not frivolous, which includes a good faith argument for an extension, modification or reversal of existing law. A lawyer for the defendant in a criminal proceeding, or the respondent in a proceeding that could result in incarceration, may nevertheless so defend the proceeding as to require that every element of the case be established.

Rule 3.5: Impartiality & Decorum of the Tribunal

Advocate A lawyer shall not: (a) seek to influence a judge, juror, prospective juror or other official by means prohibited by law; (b) communicate ex parte with such a person during the proceeding unless authorized to do so by law or court order; (c) communicate with a juror or prospective juror after discharge of the jury if: (1) the communication is prohibited by law or court order; (2) the juror has made known to the lawyer a desire not to communicate; or (3) the communication involves misrepresentation, coercion, duress or harassment; or (d) engage in conduct intended to disrupt a tribunal.

Rule 3.4: Fairness to Opposing Party & Counsel

Advocate A lawyer shall not: (a) unlawfully obstruct another party' s access to evidence or unlawfully alter, destroy or conceal a document or other material having potential evidentiary value. A lawyer shall not counsel or assist another person to do any such act; (b) falsify evidence, counsel or assist a witness to testify falsely, or offer an inducement to a witness that is prohibited by law; (c) knowingly disobey an obligation under the rules of a tribunal except for an open refusal based on an assertion that no valid obligation exists; (d) in pretrial procedure, make a frivolous discovery request or fail to make reasonably diligent effort to comply with a legally proper discovery request by an opposing party; (e) in trial, allude to any matter that the lawyer does not reasonably believe is relevant or that will not be supported by admissible evidence, assert personal knowledge of facts in issue except when testifying as a witness, or state a personal opinion as to the justness of a cause, the credibility of a witness, the culpability of a civil litigant or the guilt or innocence of an accused; or (f) request a person other than a client to refrain from voluntarily giving relevant information to another party unless: (1) the person is a relative or an employee or other agent of a client; and (2) the lawyer reasonably believes that the person's interests will not be adversely affected by refraining from giving such information.

Rule 3.8: Special Responsibilities of a Prosecutor

Advocate The prosecutor in a criminal case shall: (a) refrain from prosecuting a charge that the prosecutor knows is not supported by probable cause; (b) make reasonable efforts to assure that the accused has been advised of the right to, and the procedure for obtaining, counsel and has been given reasonable opportunity to obtain counsel; (c) not seek to obtain from an unrepresented accused a waiver of important pretrial rights, such as the right to a preliminary hearing; (d) make timely disclosure to the defense of all evidence or information known to the prosecutor that tends to negate the guilt of the accused or mitigates the offense, and, in connection with sentencing, disclose to the defense and to the tribunal all unprivileged mitigating information known to the prosecutor, except when the prosecutor is relieved of this responsibility by a protective order of the tribunal; (e) not subpoena a lawyer in a grand jury or other criminal proceeding to present evidence about a past or present client unless the prosecutor reasonably believes: (1) the information sought is not protected from disclosure by any applicable privilege; (2) the evidence sought is essential to the successful completion of an ongoing investigation or prosecution; and (3) there is no other feasible alternative to obtain the information; (f) except for statements that are necessary to inform the public of the nature and extent of the prosecutor's action and that serve a legitimate law enforcement purpose, refrain from making extrajudicial comments that have a substantial likelihood of heightening public condemnation of the accused and exercise reasonable care to prevent investigators, law enforcement personnel, employees or other persons assisting or associated with the prosecutor in a criminal case from making an extrajudicial statement that the prosecutor would be prohibited from making under Rule 3.6 or this Rule. (g) When a prosecutor knows of new, credible and material evidence creating a reasonable likelihood that a convicted defendant did not commit an offense of which the defendant was convicted, the prosecutor shall: (1) promptly disclose that evidence to an appropriate court or authority, and (2) if the conviction was obtained in the prosecutor's jurisdiction, (i) promptly disclose that evidence to the defendant unless a court authorizes delay, and (ii) undertake further investigation, or make reasonable efforts to cause an investigation, to determine whether the defendant was convicted of an offense that the defendant did not commit. (h) When a prosecutor knows of clear and convincing evidence establishing that a defendant in the prosecutor's jurisdiction was convicted of an offense that the defendant did not commit, the prosecutor shall seek to remedy the conviction.

Which of the following payments to a witness is most problematic?

Attorney C pays an expert witness a contingency fee for testifying.

Attorney Abby is representing an individual consumer in a products liability lawsuit against a manufacturer. Another attorney is representing the manufacturer in the lawsuit. Which of the following individuals may Attorney Abby directly contact to discuss the allegations in this case, without seeking the consent of the manufacturer's attorney?

An engineer who designed the product at issue for the manufacturer, but who no longer works for the manufacturer.

Attorney Anna receives a voicemail from the opposing counsel in a lawsuit. The voicemail states, "Hello, Anna. This is opposing counsel Oscar in the pending lawsuit. Please call me when you get a chance to talk about the case. Goodbye." Then, the recording has a small clicking sound, but the recording on Anna's voicemail system continues with Oscar speaking to his paralegal about strategy for this pending lawsuit. Oscar thought he had properly hung up the phone after calling Anna, but he did not. As a result, Anna's voicemail system captured about 5 minutes of Oscar's private conversation with his paralegal about confidential strategy for the case. Which of the following is correct?

Anna must promptly notify Oscar about the recording of his private conversation about strategy on her voicemail.

Attorney Ashley works in a law firm, and she is defending client Cameron in connection with high-profile criminal matter. Which of the following is correct?

Another lawyer from Attorney Ashley's firm may have a sexual relationship with Cameron as long as the other lawyer is not involved in the representation of Cameron.

Attorney Abby, who recently opened up a law practice in town, approaches Chris, a stranger, in person to inform Chris that she recently opened a law practice nearby involving family law. She asks Chris to keep her in mind if he needs legal services in the future. Which of the following is correct?

Attorney Abby is not subject to discipline if a significant motive in approaching Chris is financial gain.

During a criminal trial for burglary, Attorney Abby is representing the defendant, and in connection with sentencing, the defendant tells the judge that he has no prior criminal record. However, in fact, Attorney Abby's client does have a lengthy criminal record, which will significantly impact the sentencing in this case. If the case is still ongoing, which of the following best describes Attorney Abby's ethical duties?

Attorney Abby must take remedial measures, including, if necessary, disclosure of her client's prior record to the court.

Attorney Agatha is representing a personal injury victim for a 30% contingency fee. The case settles for $10,000, and Attorney Agatha receives a settlement check for $10,000, which she promptly deposits in her firm's client trust account. However, Attorney Agatha's client contests the 30% fee as too high because the case settled very quickly. Attorney Agatha's client believes Attorney Agatha should receive only $1,000 as her legal fee, instead of the $3,000 representing 30% of the $10,000 settlement. How should Attorney Agatha handle the amount in dispute?

Attorney Agatha must promptly disburse $7,000 to her client, keep $2,000 in the client trust account until the dispute is resolved, and transfer $1,000 from the client trust account to her law firm's bank account

Attorney Alexa used to work for the state government environmental protection agency, and as a government attorney, she investigated the Otter Oil Company for an oil spill . After leaving government service, she joins the ABC Law Firm (a popular firm in our hypos), which is currently representing Pete in a lawsuit against the Otter Oil Company for damages to Pete's fishing business arising out of the same oil spill. Which of the following is correct?

Attorney Alexa may represent Pete if the state agency gives informed consent, confirmed in writing.

Lawyer Logan, who currently works at the ABC Law Firm, owns a part owner of a local bank. A customer of the bank wants to hire Attorney Ally, who works at the ABC Law Firm, to sue the bank for a data breach involving the customer's social security number. Which of the following is correct?

Attorney Ally may represent the customer as long as Lawyer Logan's ownership interest does not cause a significant risk of materially limiting the representation of the customer.

Lawyer Logan, while working at the ABC Law Firm, formerly represented a local bank in connection with a data breach involving customer social security numbers from two years ago, and the representation of the bank is over. Lawyer Logan has left the ABC Law Firm. A customer of the bank wants to hire Attorney Ally, who works at the ABC Law Firm, to sue the bank for the same data breach. Which of the following is correct?

Attorney Ally may represent the customer as long as no one at the ABC Law Firm has any confidences material to the new matter.

Lawyer Logan, while working at the ABC Law Firm, formerly represented a local bank in connection with a data breach involving customer social security numbers from two years ago, and the representation of the bank is over. Lawyer Logan still works at the ABC Law Firm. A customer of the bank wants to hire Attorney Ally, who also works at the same ABC Law Firm, to sue the bank for the same data breach. Which of the following is correct?

Attorney Ally may represent the customer if the bank gives informed consent, confirmed in writing.

Attorney Alvin is admitted only in State A. He is hired by a client from State A to file a negligence lawsuit in court in State B. The lawsuit is not pending yet, but in preparation for the lawsuit, Attorney Alvin interviews a potential witness in State C. During the interview occurring in State C, Attorney Alvin makes a questionable statement about the case, which could be unethical in some jurisdictions. Which of the following is correct?

Attorney Alvin can be disciplined in State A, State B, and State C.

Attorney Alvin is admitted only in State A. He is hired by a client from State A to file a negligence lawsuit in court in State B. After the lawsuit is filed, Attorney Alvin interviews a potential witness in State C. During the interview occurring in State C, Attorney Alvin makes a questionable statement about the case, which could be unethical in some jurisdictions. Which of the following is correct?

Attorney Alvin can be disciplined in State A, State B, and State C; Attorney Alvin is subject to the ethics rules of State B.

Attorney Art receives a settlement offer of $10,000 for his client Chris. Without telling Chris, Attorney Art rejects the settlement offer because he thinks the case is worth more. Attorney Art then receives a $15,000 settlement offer for his client Chris, and Attorney Art immediately informs Chris of the $15,000 offer, which Chris gladly accepts. Which of the following is correct?

Attorney Art is subject to discipline for rejecting the $10,000 offer.

Attorney Art, a solo practitioner, is representing a client pursuant to a 30% contingency fee against a local store for a car accident between his client and the store's delivery truck. During the representation, a complex issue of bankruptcy law arises when the store files for bankruptcy in the middle of the lawsuit, and Attorney Art asks Attorney Belinda, an expert in bankruptcy law, to assist with the case. Assuming that the 30% contingency fee is reasonable and the client provides informed consent, confirmed in writing, may Attorney Art split his 30% contingency fee with Attorney Belinda?

Attorney Art may split the fee in half with Attorney Belinda as long as each lawyer does half the legal work for the representation.; Attorney Art may give 10% of the contingency fee to Attorney Belinda, with Attorney Art keeping 90% of the contingency fee, as long as each lawyer assumes joint responsibility for the representation.

Client Calvin, who is under indictment for murder, is represented by Attorney Art. In the course of the representation, Calvin confesses to Attorney Art that he had previously killed two other persons. Attorney Art, with Calvin's consent, makes a tape recording of Calvin's confession regarding the other murders in order to possibly use this confession as the basis for an insanity defense at the future trial. The two other murders have remained unsolved, and their bodies have not been recovered. Neither the police nor prosecutors suspect that Calvin committed the two other murders, and the two other murder victims are suspected to be merely missing and not dead. Which of the following is correct?

Attorney Art must not disclose the information about the other murders because this information is confidential, unless the client gives informed consent.

Attorney Ava is admitted only in State A. She is hired by a client from State A to file a theft of trade secrets lawsuit in court in State B. The lawsuit is not pending yet. To help prepare for the lawsuit, Attorney Ava reviews some documents and files located in State C because these documents are likely to be used as evidence in the lawsuit. Which of the following is correct?

Attorney Ava may review the documents in State C if she reasonably expects to be authorized to appear in the State B proceeding.

Which of the following attorneys best fulfilled his or her duty to provide pro bono services?

Attorney B, who during the year provides 50 hours of legal services, without charging a fee, to assist a local public elementary school in satisfying the legal requirements for a government-sponsored free meal program for students below the poverty level.

Lawyer Lisa is representing client Chris in connection a civil battery claim arising from a bar fight. The other person involved in the fight is suing Chris for battery. Which of the following pieces of information is protected by the attorney's duty of confidentiality?

Chris tells Lawyer Lisa that the plaintiff threw a bottle at him; The bartender tells Lawyer Lisa that the plaintiff threw a bottle at Chris; Lawyer Lisa reads a newspaper story about the bar fight, stating that an eyewitness saw the plaintiff throw the first punch in the fight.

In which of the following circumstances may a lawyer reveal information normally protected by the duty of confidentiality?

Client B hires Lawyer B to represent him in negotiating the sale of a company, and right before the sale, Client B confesses to Lawyer B that the business deal is fraudulent and will cause serious financial harm to the purchasing party because the company's profits and assets have been grossly misrepresented.

Which of the following lawyers are subject to discipline?

Client Carl suggests to Attorney Christine that he should hire someone to threaten the judge in the case by slashing the judge's tires. Attorney Christine ignores Carl's suggestion, without telling Carl anything about his improper suggestion or following through with the improper suggestion.

Rule 1.13: Organization as Client

Client-Lawyer Relationship (a) A lawyer employed or retained by an organization represents the organization acting through its duly authorized constituents. (b) If a lawyer for an organization knows that an officer, employee or other person associated with the organization is engaged in action, intends to act or refuses to act in a matter related to the representation that is a violation of a legal obligation to the organization, or a violation of law that reasonably might be imputed to the organization, and that is likely to result in substantial injury to the organization, then the lawyer shall proceed as is reasonably necessary in the best interest of the organization. Unless the lawyer reasonably believes that it is not necessary in the best interest of the organization to do so, the lawyer shall refer the matter to higher authority in the organization, including, if warranted by the circumstances to the highest authority that can act on behalf of the organization as determined by applicable law. (c) Except as provided in paragraph (d), if (1) despite the lawyer's efforts in accordance with paragraph (b) the highest authority that can act on behalf of the organization insists upon or fails to address in a timely and appropriate manner an action, or a refusal to act, that is clearly a violation of law, and (2) the lawyer reasonably believes that the violation is reasonably certain to result in substantial injury to the organization, then the lawyer may reveal information relating to the representation whether or not Rule 1.6 permits such disclosure, but only if and to the extent the lawyer reasonably believes necessary to prevent substantial injury to the organization. (d) Paragraph (c) shall not apply with respect to information relating to a lawyer's representation of an organization to investigate an alleged violation of law, or to defend the organization or an officer, employee or other constituent associated with the organization against a claim arising out of an alleged violation of law. (e) A lawyer who reasonably believes that he or she has been discharged because of the lawyer's actions taken pursuant to paragraphs (b) or (c), or who withdraws under circumstances that require or permit the lawyer to take action under either of those paragraphs, shall proceed as the lawyer reasonably believes necessary to assure that the organization's highest authority is informed of the lawyer's discharge or withdrawal. (f) In dealing with an organization's directors, officers, employees, members, shareholders or other constituents, a lawyer shall explain the identity of the client when the lawyer knows or reasonably should know that the organization's interests are adverse to those of the constituents with whom the lawyer is dealing. (g) A lawyer representing an organization may also represent any of its directors, officers, employees, members, shareholders or other constituents, subject to the provisions of Rule 1.7. If the organization's consent to the dual representation is required by Rule 1.7, the consent shall be given by an appropriate official of the organization other than the individual who is to be represented, or by the shareholders.

Rule 2.3: Evaluation for Use by Third Persons

Counselor (a) A lawyer may provide an evaluation of a matter affecting a client for the use of someone other than the client if the lawyer reasonably believes that making the evaluation is compatible with other aspects of the lawyer's relationship with the client. (b) When the lawyer knows or reasonably should know that the evaluation is likely to affect the client's interests materially and adversely, the lawyer shall not provide the evaluation unless the client gives informed consent. (c) Except as disclosure is authorized in connection with a report of an evaluation, information relating to the evaluation is otherwise protected by Rule 1.6.

Rule 1.15: Safekeeping Property

Client-Lawyer Relationship (a) A lawyer shall hold property of clients or third persons that is in a lawyer's possession in connection with a representation separate from the lawyer's own property. Funds shall be kept in a separate account maintained in the state where the lawyer's office is situated, or elsewhere with the consent of the client or third person. Other property shall be identified as such and appropriately safeguarded. Complete records of such account funds and other property shall be kept by the lawyer and shall be preserved for a period of [five years] after termination of the representation. (b) A lawyer may deposit the lawyer's own funds in a client trust account for the sole purpose of paying bank service charges on that account, but only in an amount necessary for that purpose. (c) A lawyer shall deposit into a client trust account legal fees and expenses that have been paid in advance, to be withdrawn by the lawyer only as fees are earned or expenses incurred. (d) Upon receiving funds or other property in which a client or third person has an interest, a lawyer shall promptly notify the client or third person. Except as stated in this rule or otherwise permitted by law or by agreement with the client, a lawyer shall promptly deliver to the client or third person any funds or other property that the client or third person is entitled to receive and, upon request by the client or third person, shall promptly render a full accounting regarding such property. (e) When in the course of representation a lawyer is in possession of property in which two or more persons (one of whom may be the lawyer) claim interests, the property shall be kept separate by the lawyer until the dispute is resolved. The lawyer shall promptly distribute all portions of the property as to which the interests are not in dispute.

Rule 1.8: Current Clients: Specific Rules

Client-Lawyer Relationship (a) A lawyer shall not enter into a business transaction with a client or knowingly acquire an ownership, possessory, security or other pecuniary interest adverse to a client unless: (1) the transaction and terms on which the lawyer acquires the interest are fair and reasonable to the client and are fully disclosed and transmitted in writing in a manner that can be reasonably understood by the client; (2) the client is advised in writing of the desirability of seeking and is given a reasonable opportunity to seek the advice of independent legal counsel on the transaction; and (3) the client gives informed consent, in a writing signed by the client, to the essential terms of the transaction and the lawyer's role in the transaction, including whether the lawyer is representing the client in the transaction. (b) A lawyer shall not use information relating to representation of a client to the disadvantage of the client unless the client gives informed consent, except as permitted or required by these Rules. (c) A lawyer shall not solicit any substantial gift from a client, including a testamentary gift, or prepare on behalf of a client an instrument giving the lawyer or a person related to the lawyer any substantial gift unless the lawyer or other recipient of the gift is related to the client. For purposes of this paragraph, related persons include a spouse, child, grandchild, parent, grandparent or other relative or individual with whom the lawyer or the client maintains a close, familial relationship. (d) Prior to the conclusion of representation of a client, a lawyer shall not make or negotiate an agreement giving the lawyer literary or media rights to a portrayal or account based in substantial part on information relating to the representation. (e) A lawyer shall not provide financial assistance to a client in connection with pending or contemplated litigation, except that: (1) a lawyer may advance court costs and expenses of litigation, the repayment of which may be contingent on the outcome of the matter; and (2) a lawyer representing an indigent client may pay court costs and expenses of litigation on behalf of the client. (f) A lawyer shall not accept compensation for representing a client from one other than the client unless: (1) the client gives informed consent; (2) there is no interference with the lawyer's independence of professional judgment or with the client-lawyer relationship; and (3) information relating to representation of a client is protected as required by Rule 1.6. (g) A lawyer who represents two or more clients shall not participate in making an aggregate settlement of the claims of or against the clients, or in a criminal case an aggregated agreement as to guilty or nolo contendere pleas, unless each client gives informed consent, in a writing signed by the client. The lawyer's disclosure shall include the existence and nature of all the claims or pleas involved and of the participation of each person in the settlement. (h) A lawyer shall not: (1) make an agreement prospectively limiting the lawyer's liability to a client for malpractice unless the client is independently represented in making the agreement; or (2) settle a claim or potential claim for such liability with an unrepresented client or former client unless that person is advised in writing of the desirability of seeking and is given a reasonable opportunity to seek the advice of independent legal counsel in connection therewith. (i) A lawyer shall not acquire a proprietary interest in the cause of action or subject matter of litigation the lawyer is conducting for a client, except that the lawyer may: (1) acquire a lien authorized by law to secure the lawyer's fee or expenses; and (2) contract with a client for a reasonable contingent fee in a civil case. (j) A lawyer shall not have sexual relations with a client unless a consensual sexual relationship existed between them when the client-lawyer relationship commenced. (k) While lawyers are associated in a firm, a prohibition in the foregoing paragraphs (a) through (i) that applies to any one of them shall apply to all of them.

Rule 1.5: Fees

Client-Lawyer Relationship (a) A lawyer shall not make an agreement for, charge, or collect an unreasonable fee or an unreasonable amount for expenses. The factors to be considered in determining the reasonableness of a fee include the following: (1) the time and labor required, the novelty and difficulty of the questions involved, and the skill requisite to perform the legal service properly; (2) the likelihood, if apparent to the client, that the acceptance of the particular employment will preclude other employment by the lawyer; (3) the fee customarily charged in the locality for similar legal services; (4) the amount involved and the results obtained; (5) the time limitations imposed by the client or by the circumstances; (6) the nature and length of the professional relationship with the client; (7) the experience, reputation, and ability of the lawyer or lawyers performing the services; and (8) whether the fee is fixed or contingent. (b) The scope of the representation and the basis or rate of the fee and expenses for which the client will be responsible shall be communicated to the client, preferably in writing, before or within a reasonable time after commencing the representation, except when the lawyer will charge a regularly represented client on the same basis or rate. Any changes in the basis or rate of the fee or expenses shall also be communicated to the client. (c) A fee may be contingent on the outcome of the matter for which the service is rendered, except in a matter in which a contingent fee is prohibited by paragraph (d) or other law. A contingent fee agreement shall be in a writing signed by the client and shall state the method by which the fee is to be determined, including the percentage or percentages that shall accrue to the lawyer in the event of settlement, trial or appeal; litigation and other expenses to be deducted from the recovery; and whether such expenses are to be deducted before or after the contingent fee is calculated. The agreement must clearly notify the client of any expenses for which the client will be liable whether or not the client is the prevailing party. Upon conclusion of a contingent fee matter, the lawyer shall provide the client with a written statement stating the outcome of the matter and, if there is a recovery, showing the remittance to the client and the method of its determination. (d) A lawyer shall not enter into an arrangement for, charge, or collect: (1) any fee in a domestic relations matter, the payment or amount of which is contingent upon the securing of a divorce or upon the amount of alimony or support, or property settlement in lieu thereof; or (2) a contingent fee for representing a defendant in a criminal case. (e) A division of a fee between lawyers who are not in the same firm may be made only if: (1) the division is in proportion to the services performed by each lawyer or each lawyer assumes joint responsibility for the representation; (2) the client agrees to the arrangement, including the share each lawyer will receive, and the agreement is confirmed in writing; and (3) the total fee is reasonable.

Rule 1.6: Confidentiality of Information

Client-Lawyer Relationship (a) A lawyer shall not reveal information relating to the representation of a client unless the client gives informed consent, the disclosure is impliedly authorized in order to carry out the representation or the disclosure is permitted by paragraph (b). (b) A lawyer may reveal information relating to the representation of a client to the extent the lawyer reasonably believes necessary: (1) to prevent reasonably certain death or substantial bodily harm; (2) to prevent the client from committing a crime or fraud that is reasonably certain to result in substantial injury to the financial interests or property of another and in furtherance of which the client has used or is using the lawyer's services; (3) to prevent, mitigate or rectify substantial injury to the financial interests or property of another that is reasonably certain to result or has resulted from the client's commission of a crime or fraud in furtherance of which the client has used the lawyer's services; (4) to secure legal advice about the lawyer's compliance with these Rules; (5) to establish a claim or defense on behalf of the lawyer in a controversy between the lawyer and the client, to establish a defense to a criminal charge or civil claim against the lawyer based upon conduct in which the client was involved, or to respond to allegations in any proceeding concerning the lawyer's representation of the client; (6) to comply with other law or a court order; or (7) to detect and resolve conflicts of interest arising from the lawyer's change of employment or from changes in the composition or ownership of a firm, but only if the revealed information would not compromise the attorney-client privilege or otherwise prejudice the client. (c) A lawyer shall make reasonable efforts to prevent the inadvertent or unauthorized disclosure of, or unauthorized access to, information relating to the representation of a client.

Rule 1.4: Communications

Client-Lawyer Relationship (a) A lawyer shall: (1) promptly inform the client of any decision or circumstance with respect to which the client's informed consent, as defined in Rule 1.0(e), is required by these Rules; (2) reasonably consult with the client about the means by which the client's objectives are to be accomplished; (3) keep the client reasonably informed about the status of the matter; (4) promptly comply with reasonable requests for information; and (5) consult with the client about any relevant limitation on the lawyer's conduct when the lawyer knows that the client expects assistance not permitted by the Rules of Professional Conduct or other law. (b) A lawyer shall explain a matter to the extent reasonably necessary to permit the client to make informed decisions regarding the representation.

Rule 1.9: Duties to Former Clients

Client-Lawyer Relationship (a) A lawyer who has formerly represented a client in a matter shall not thereafter represent another person in the same or a substantially related matter in which that person's interests are materially adverse to the interests of the former client unless the former client gives informed consent, confirmed in writing. (b) A lawyer shall not knowingly represent a person in the same or a substantially related matter in which a firm with which the lawyer formerly was associated had previously represented a client (1) whose interests are materially adverse to that person; and (2) about whom the lawyer had acquired information protected by Rules 1.6 and 1.9(c) that is material to the matter; unless the former client gives informed consent, confirmed in writing. (c) A lawyer who has formerly represented a client in a matter or whose present or former firm has formerly represented a client in a matter shall not thereafter: (1) use information relating to the representation to the disadvantage of the former client except as these Rules would permit or require with respect to a client, or when the information has become generally known; or (2) reveal information relating to the representation except as these Rules would permit or require with respect to a client.

Rule 1.18: Duties to Prospective Client

Client-Lawyer Relationship (a) A person who consults with a lawyer about the possibility of forming a client-lawyer relationship with respect to a matter is a prospective client. (b) Even when no client-lawyer relationship ensues, a lawyer who has learned information from a prospective client shall not use or reveal that information, except as Rule 1.9 would permit with respect to information of a former client. (c) A lawyer subject to paragraph (b) shall not represent a client with interests materially adverse to those of a prospective client in the same or a substantially related matter if the lawyer received information from the prospective client that could be significantly harmful to that person in the matter, except as provided in paragraph (d). If a lawyer is disqualified from representation under this paragraph, no lawyer in a firm with which that lawyer is associated may knowingly undertake or continue representation in such a matter, except as provided in paragraph (d). (d) When the lawyer has received disqualifying information as defined in paragraph (c), representation is permissible if: (1) both the affected client and the prospective client have given informed consent, confirmed in writing, or: (2) the lawyer who received the information took reasonable measures to avoid exposure to more disqualifying information than was reasonably necessary to determine whether to represent the prospective client; and (i) the disqualified lawyer is timely screened from any participation in the matter and is apportioned no part of the fee therefrom; and (ii) written notice is promptly given to the prospective client.

Rule 1.11: Special Conflicts of Interest for Former & Current Government Officers & Employees

Client-Lawyer Relationship (a) Except as law may otherwise expressly permit, a lawyer who has formerly served as a public officer or employee of the government: (1) is subject to Rule 1.9(c); and (2) shall not otherwise represent a client in connection with a matter in which the lawyer participated personally and substantially as a public officer or employee, unless the appropriate government agency gives its informed consent, confirmed in writing, to the representation. (b) When a lawyer is disqualified from representation under paragraph (a), no lawyer in a firm with which that lawyer is associated may knowingly undertake or continue representation in such a matter unless: (1) the disqualified lawyer is timely screened from any participation in the matter and is apportioned no part of the fee therefrom; and (2) written notice is promptly given to the appropriate government agency to enable it to ascertain compliance with the provisions of this rule. (c) Except as law may otherwise expressly permit, a lawyer having information that the lawyer knows is confidential government information about a person acquired when the lawyer was a public officer or employee, may not represent a private client whose interests are adverse to that person in a matter in which the information could be used to the material disadvantage of that person. As used in this Rule, the term "confidential government information" means information that has been obtained under governmental authority and which, at the time this Rule is applied, the government is prohibited by law from disclosing to the public or has a legal privilege not to disclose and which is not otherwise available to the public. A firm with which that lawyer is associated may undertake or continue representation in the matter only if the disqualified lawyer is timely screened from any participation in the matter and is apportioned no part of the fee therefrom. (d) Except as law may otherwise expressly permit, a lawyer currently serving as a public officer or employee: (1) is subject to Rules 1.7 and 1.9; and (2) shall not: (i) participate in a matter in which the lawyer participated personally and substantially while in private practice or nongovernmental employment, unless the appropriate government agency gives its informed consent, confirmed in writing; or (ii) negotiate for private employment with any person who is involved as a party or as lawyer for a party in a matter in which the lawyer is participating personally and substantially, except that a lawyer serving as a law clerk to a judge, other adjudicative officer or arbitrator may negotiate for private employment as permitted by Rule 1.12(b) and subject to the conditions stated in Rule 1.12(b). (e) As used in this Rule, the term "matter" includes: (1) any judicial or other proceeding, application, request for a ruling or other determination, contract, claim, controversy, investigation, charge, accusation, arrest or other particular matter involving a specific party or parties, and (2) any other matter covered by the conflict of interest rules of the appropriate government agency.

Rule 1.7: Conflict of Interest: Current Clients

Client-Lawyer Relationship (a) Except as provided in paragraph (b), a lawyer shall not represent a client if the representation involves a concurrent conflict of interest. A concurrent conflict of interest exists if: (1) the representation of one client will be directly adverse to another client; or (2) there is a significant risk that the representation of one or more clients will be materially limited by the lawyer's responsibilities to another client, a former client or a third person or by a personal interest of the lawyer. (b) Notwithstanding the existence of a concurrent conflict of interest under paragraph (a), a lawyer may represent a client if: (1) the lawyer reasonably believes that the lawyer will be able to provide competent and diligent representation to each affected client; (2) the representation is not prohibited by law; (3) the representation does not involve the assertion of a claim by one client against another client represented by the lawyer in the same litigation or other proceeding before a tribunal; and (4) each affected client gives informed consent, confirmed in writing.

Rule 1.16: Declining or Terminating Representation

Client-Lawyer Relationship (a) Except as stated in paragraph (c), a lawyer shall not represent a client or, where representation has commenced, shall withdraw from the representation of a client if: Mandatory language (1) the representation will result in violation of the rules of professional conduct or other law; (2) the lawyer's physical or mental condition materially impairs the lawyer's ability to represent the client; or (3) the lawyer is discharged. (b) Except as stated in paragraph (c), a lawyer may withdraw from representing a client if: gives discretion (1) withdrawal can be accomplished without material adverse effect on the interests of the client; (2) the client persists in a course of action involving the lawyer's services that the lawyer reasonably believes is criminal or fraudulent; (3) the client has used the lawyer's services to perpetrate a crime or fraud; (4) the client insists upon taking action that the lawyer considers repugnant or with which the lawyer has a fundamental disagreement; (5) the client fails substantially to fulfill an obligation to the lawyer regarding the lawyer's services and has been given reasonable warning that the lawyer will withdraw unless the obligation is fulfilled; (6) the representation will result in an unreasonable financial burden on the lawyer or has been rendered unreasonably difficult by the client; or (7) other good cause for withdrawal exists. (c) A lawyer must comply with applicable law requiring notice to or permission of a tribunal when terminating a representation. When ordered to do so by a tribunal, a lawyer shall continue representation notwithstanding good cause for terminating the representation. Exceptions that override both sections (a) and (b) (d) Upon termination of representation, a lawyer shall take steps to the extent reasonably practicable to protect a client's interests, such as giving reasonable notice to the client, allowing time for employment of other counsel, surrendering papers and property to which the client is entitled and refunding any advance payment of fee or expense that has not been earned or incurred. The lawyer may retain papers relating to the client to the extent permitted by other law.

Rule 1.12: Former Judge, Arbitrator, Mediator or Other Third-Party Neutral

Client-Lawyer Relationship (a) Except as stated in paragraph (d), a lawyer shall not represent anyone in connection with a matter in which the lawyer participated personally and substantially as a judge or other adjudicative officer or law clerk to such a person or as an arbitrator, mediator or other third-party neutral, unless all parties to the proceeding give informed consent, confirmed in writing. (b) A lawyer shall not negotiate for employment with any person who is involved as a party or as lawyer for a party in a matter in which the lawyer is participating personally and substantially as a judge or other adjudicative officer or as an arbitrator, mediator or other third-party neutral. A lawyer serving as a law clerk to a judge or other adjudicative officer may negotiate for employment with a party or lawyer involved in a matter in which the clerk is participating personally and substantially, but only after the lawyer has notified the judge or other adjudicative officer. (c) If a lawyer is disqualified by paragraph (a), no lawyer in a firm with which that lawyer is associated may knowingly undertake or continue representation in the matter unless: (1) the disqualified lawyer is timely screened from any participation in the matter and is apportioned no part of the fee therefrom; and (2) written notice is promptly given to the parties and any appropriate tribunal to enable them to ascertain compliance with the provisions of this Rule. (d) An arbitrator selected as a partisan of a party in a multimember arbitration panel is not prohibited from subsequently representing that party.

Rule 1.2: Scope of Representation & Allocation of Authority Between Client & Lawyer

Client-Lawyer Relationship (a) Subject to paragraphs (c) and (d), a lawyer shall abide by a client's decisions concerning the objectives of representation and, as required by Rule 1.4, shall consult with the client as to the means by which they are to be pursued. A lawyer may take such action on behalf of the client as is impliedly authorized to carry out the representation. A lawyer shall abide by a client's decision whether to settle a matter. In a criminal case, the lawyer shall abide by the client's decision, after consultation with the lawyer, as to a plea to be entered, whether to waive jury trial and whether the client will testify. (b) A lawyer's representation of a client, including representation by appointment, does not constitute an endorsement of the client's political, economic, social or moral views or activities. (c) A lawyer may limit the scope of the representation if the limitation is reasonable under the circumstances and the client gives informed consent. (d) A lawyer shall not counsel a client to engage, or assist a client, in conduct that the lawyer knows is criminal or fraudulent, but a lawyer may discuss the legal consequences of any proposed course of conduct with a client and may counsel or assist a client to make a good faith effort to determine the validity, scope, meaning or application of the law.

Rule 1.14: Client with Diminished Capacity

Client-Lawyer Relationship (a) When a client's capacity to make adequately considered decisions in connection with a representation is diminished, whether because of minority, mental impairment or for some other reason, the lawyer shall, as far as reasonably possible, maintain a normal client-lawyer relationship with the client. (b) When the lawyer reasonably believes that the client has diminished capacity, is at risk of substantial physical, financial or other harm unless action is taken and cannot adequately act in the client's own interest, the lawyer may take reasonably necessary protective action, including consulting with individuals or entities that have the ability to take action to protect the client and, in appropriate cases, seeking the appointment of a guardian ad litem, conservator or guardian. (c) Information relating to the representation of a client with diminished capacity is protected by Rule 1.6. When taking protective action pursuant to paragraph (b), the lawyer is impliedly authorized under Rule 1.6(a) to reveal information about the client, but only to the extent reasonably necessary to protect the client's interests.

Rule 1.10: Imputation of Conflicts of Interest: General Rule

Client-Lawyer Relationship (a) While lawyers are associated in a firm, none of them shall knowingly represent a client when any one of them practicing alone would be prohibited from doing so by Rules 1.7 or 1.9, unless (1) the prohibition is based on a personal interest of the disqualified lawyer and does not present a significant risk of materially limiting the representation of the client by the remaining lawyers in the firm; or (2) the prohibition is based upon Rule 1.9(a) or (b) and arises out of the disqualified lawyer's association with a prior firm, and (i) the disqualified lawyer is timely screened from any participation in the matter and is apportioned no part of the fee therefrom; (ii) written notice is promptly given to any affected former client to enable the former client to ascertain compliance with the provisions of this Rule, which shall include a description of the screening procedures employed; a statement of the firm's and of the screened lawyer's compliance with these Rules; a statement that review may be available before a tribunal; and an agreement by the firm to respond promptly to any written inquiries or objections by the former client about the screening procedures; and (iii) certifications of compliance with these Rules and with the screening procedures are provided to the former client by the screened lawyer and by a partner of the firm, at reasonable intervals upon the former client's written request and upon termination of the screening procedures. (b) When a lawyer has terminated an association with a firm, the firm is not prohibited from thereafter representing a person with interests materially adverse to those of a client represented by the formerly associated lawyer and not currently represented by the firm, unless: (1) the matter is the same or substantially related to that in which the formerly associated lawyer represented the client; and (2) any lawyer remaining in the firm has information protected by Rules 1.6 and 1.9(c) that is material to the matter. (c) A disqualification prescribed by this rule may be waived by the affected client under the conditions stated in Rule 1.7. (d) The disqualification of lawyers associated in a firm with former or current government lawyers is governed by Rule 1.11.

Rule 1.17: Sale of Law Practice

Client-Lawyer Relationship A lawyer or a law firm may sell or purchase a law practice, or an area of law practice, including good will, if the following conditions are satisfied: (a) The seller ceases to engage in the private practice of law, or in the area of practice that has been sold, [in the geographic area] [in the jurisdiction] (a jurisdiction may elect either version) in which the practice has been conducted; (b) The entire practice, or the entire area of practice, is sold to one or more lawyers or law firms; (c) The seller gives written notice to each of the seller's clients regarding: (1) the proposed sale; (2) the client's right to retain other counsel or to take possession of the file; and (3) the fact that the client's consent to the transfer of the client's files will be presumed if the client does not take any action or does not otherwise object within ninety (90) days of receipt of the notice. If a client cannot be given notice, the representation of that client may be transferred to the purchaser only upon entry of an order so authorizing by a court having jurisdiction. The seller may disclose to the court in camera information relating to the representation only to the extent necessary to obtain an order authorizing the transfer of a file. (d) The fees charged clients shall not be increased by reason of the sale.

Rule 1.3: Diligence

Client-Lawyer Relationship A lawyer shall act with reasonable diligence and promptness in representing a client.

Rule 1.1: Competence

Client-Lawyer Relationship A lawyer shall provide competent representation to a client. Competent representation requires the legal knowledge, skill, thoroughness and preparation reasonably necessary for the representation

Rule 2.4: Lawyer Serving as Third-Party Neutral

Counselor (a) A lawyer serves as a third-party neutral when the lawyer assists two or more persons who are not clients of the lawyer to reach a resolution of a dispute or other matter that has arisen between them. Service as a third-party neutral may include service as an arbitrator, a mediator or in such other capacity as will enable the lawyer to assist the parties to resolve the matter. (b) A lawyer serving as a third-party neutral shall inform unrepresented parties that the lawyer is not representing them. When the lawyer knows or reasonably should know that a party does not understand the lawyer's role in the matter, the lawyer shall explain the difference between the lawyer's role as a third-party neutral and a lawyer's role as one who represents a client.

Rule 2.1: Advisor

Counselor In representing a client, a lawyer shall exercise independent professional judgment and render candid advice. In rendering advice, a lawyer may refer not only to law but to other considerations such as moral, economic, social and political factors, that may be relevant to the client's situation.

Lawyer Livia represented a client as a plaintiff in a civil trial alleging that the defendant had engaged in patent infringement. A key issue at trial is the exact date Livia's client allegedly made the invention that is the subject of the lawsuit. At trial, Livia's client testifies that he made the invention long before the defendant began making its own copycat products. The jury finds that the defendant infringed on the patent belonging to Livia's client, and Livia's client wins a jury verdict in his favor. Assume that at some point, Livia's client confesses to her that he lied on the witness stand and his testimony about the date was false because he made his invention after the defendant made its own products. Under which of the following circumstances must Lawyer Livia disclose the perjury to the court?I. Livia learns of the perjury while the jury is deliberating. II. Livia learns of the perjury after the jury has issued a verdict and the time for appeal has passed with no appeal having been taken. III. Livia learns of the perjury while the case is pending on appeal.

I and III only.

Which of the following violates an ethical rule?

Lawyer A makes a false statement about the qualifications of a current judge; Lawyer B makes a false statement of material fact on his bar admission application; Lawyer C makes a false statement about the qualifications of a candidate for election to become a judge.

Rule 5.2: Responsibilities of a Subordinate Lawyer

Law Firms And Associations (a) A lawyer is bound by the Rules of Professional Conduct notwithstanding that the lawyer acted at the direction of another person. (b) A subordinate lawyer does not violate the Rules of Professional Conduct if that lawyer acts in accordance with a supervisory lawyer's reasonable resolution of an arguable question of professional duty.

Which of the following is correct?

Lawyer A, a personal injury lawyer, pays for the costs of an expert witness for a lawsuit for a client who is indigent and involved in a car accident.

Rule 5.4: Professional Independence of a Lawyer

Law Firms And Associations (a) A lawyer or law firm shall not share legal fees with a nonlawyer, except that: (1) an agreement by a lawyer with the lawyer's firm, partner, or associate may provide for the payment of money, over a reasonable period of time after the lawyer's death, to the lawyer's estate or to one or more specified persons; (2) a lawyer who purchases the practice of a deceased, disabled, or disappeared lawyer may, pursuant to the provisions of Rule 1.17, pay to the estate or other representative of that lawyer the agreed-upon purchase price; (3) a lawyer or law firm may include nonlawyer employees in a compensation or retirement plan, even though the plan is based in whole or in part on a profit-sharing arrangement; and (4) a lawyer may share court-awarded legal fees with a nonprofit organization that employed, retained or recommended employment of the lawyer in the matter. (b) A lawyer shall not form a partnership with a nonlawyer if any of the activities of the partnership consist of the practice of law. (c) A lawyer shall not permit a person who recommends, employs, or pays the lawyer to render legal services for another to direct or regulate the lawyer's professional judgment in rendering such legal services. (d) A lawyer shall not practice with or in the form of a professional corporation or association authorized to practice law for a profit, if: (1) a nonlawyer owns any interest therein, except that a fiduciary representative of the estate of a lawyer may hold the stock or interest of the lawyer for a reasonable time during administration; (2) a nonlawyer is a corporate director or officer thereof or occupies the position of similar responsibility in any form of association other than a corporation ; or (3) a nonlawyer has the right to direct or control the professional judgment of a lawyer.

Rule 5.7: Responsibilities Regarding Law-related Services

Law Firms And Associations (a) A lawyer shall be subject to the Rules of Professional Conduct with respect to the provision of law-related services, as defined in paragraph (b), if the law-related services are provided: (1) by the lawyer in circumstances that are not distinct from the lawyer's provision of legal services to clients; or (2) in other circumstances by an entity controlled by the lawyer individually or with others if the lawyer fails to take reasonable measures to assure that a person obtaining the law-related services knows that the services are not legal services and that the protections of the client-lawyer relationship do not exist. (b) The term "law-related services" denotes services that might reasonably be performed in conjunction with and in substance are related to the provision of legal services, and that are not prohibited as unauthorized practice of law when provided by a nonlawyer.

Rule 5.5: Unauthorized Practice of Law; Multijurisdictional Practice of Law

Law Firms And Associations (a) A lawyer shall not practice law in a jurisdiction in violation of the regulation of the legal profession in that jurisdiction, or assist another in doing so. (b) A lawyer who is not admitted to practice in this jurisdiction shall not: (1) except as authorized by these Rules or other law, establish an office or other systematic and continuous presence in this jurisdiction for the practice of law; or (2) hold out to the public or otherwise represent that the lawyer is admitted to practice law in this jurisdiction. (c) A lawyer admitted in another United States jurisdiction, and not disbarred or suspended from practice in any jurisdiction, may provide legal services on a temporary basis in this jurisdiction that: MOST CHALLENGING PROVISION OF THIS RULE; know all (4) provisions (1) are undertaken in association with a lawyer who is admitted to practice in this jurisdiction and who actively participates in the matter; (2) are in or reasonably related to a pending or potential proceeding before a tribunal in this or another jurisdiction, if the lawyer, or a person the lawyer is assisting, is authorized by law or order to appear in such proceeding or reasonably expects to be so authorized; (3) are in or reasonably related to a pending or potential arbitration, mediation, or other alternative resolution proceeding in this or another jurisdiction, if the services arise out of or are reasonably related to the lawyer's practice in a jurisdiction in which the lawyer is admitted to practice and are not services for which the forum requires pro hac vice admission; or (4) are not within paragraphs (c) (2) or (c)(3) and arise out of or are reasonably related to the lawyer's practice in a jurisdiction in which the lawyer is admitted to practice. (d) A lawyer admitted in another United States jurisdiction or in a foreign jurisdiction, and not disbarred or suspended from practice in any jurisdiction or the equivalent thereof, or a person otherwise lawfully practicing as an in-house counsel under the laws of a foreign jurisdiction, may provide legal services through an office or other systematic and continuous presence in this jurisdiction that: (1) are provided to the lawyer's employer or its organizational affiliates, are not services for which the forum requires pro hac vice admission; and when performed by a foreign lawyer and requires advice on the law of this or another U.S. jurisdiction or of the United States, such advice shall be based upon the advice of a lawyer who is duly licensed and authorized by the jurisdiction to provide such advice; or (2) are services that the lawyer is authorized by federal or other law or rule to provide in this jurisdiction. (e) For purposes of paragraph (d): (1) the foreign lawyer must be a member in good standing of a recognized legal profession in a foreign jurisdiction, the members of which are admitted to practice as lawyers or counselors at law or the equivalent, and subject to effective regulation and discipline by a duly constituted professional body or a public authority; or, (2) the person otherwise lawfully practicing as an in-house counsel under the laws of a foreign jurisdiction must be authorized to practice under this Rule by, in the exercise of its discretion, [the highest court of this jurisdiction].

Rule 5.1: Responsibilities of a Partner or Supervisory Lawyer

Law Firms And Associations (a) A partner in a law firm, and a lawyer who individually or together with other lawyers possesses comparable managerial authority in a law firm, shall make reasonable efforts to ensure that the firm has in effect measures giving reasonable assurance that all lawyers in the firm conform to the Rules of Professional Conduct. (b) A lawyer having direct supervisory authority over another lawyer shall make reasonable efforts to ensure that the other lawyer conforms to the Rules of Professional Conduct. (c) A lawyer shall be responsible for another lawyer's violation of the Rules of Professional Conduct if: (1) the lawyer orders or, with knowledge of the specific conduct, ratifies the conduct involved; or (2) the lawyer is a partner or has comparable managerial authority in the law firm in which the other lawyer practices, or has direct supervisory authority over the other lawyer, and knows of the conduct at a time when its consequences can be avoided or mitigated but fails to take reasonable remedial action.

Rule 5.6: Restrictions on Rights to Practice

Law Firms And Associations A lawyer shall not participate in offering or making: (a) a partnership, shareholders, operating, employment, or other similar type of agreement that restricts the right of a lawyer to practice after termination of the relationship, except an agreement concerning benefits upon retirement; or (b) an agreement in which a restriction on the lawyer's right to practice is part of the settlement of a client controversy.

Rule 5.3: Responsibilities Regarding Nonlawyer Assistance

Law Firms And Associations With respect to a nonlawyer employed or retained by or associated with a lawyer: (a) a partner, and a lawyer who individually or together with other lawyers possesses comparable managerial authority in a law firm shall make reasonable efforts to ensure that the firm has in effect measures giving reasonable assurance that the person's conduct is compatible with the professional obligations of the lawyer; (b) a lawyer having direct supervisory authority over the nonlawyer shall make reasonable efforts to ensure that the person's conduct is compatible with the professional obligations of the lawyer; and (c) a lawyer shall be responsible for conduct of such a person that would be a violation of the Rules of Professional Conduct if engaged in by a lawyer if: (1) the lawyer orders or, with the knowledge of the specific conduct, ratifies the conduct involved; or (2) the lawyer is a partner or has comparable managerial authority in the law firm in which the person is employed, or has direct supervisory authority over the person, and knows of the conduct at a time when its consequences can be avoided or mitigated but fails to take reasonable remedial action.

Lawyer Art is admitted to practice law only in State A, and Lawyer Belle is admitted to practice law only in State B. Lawyer Belle files a wage lawsuit on behalf of a client in court in State C, where she is admitted through pro hac vice admission. Lawyer Belle needs to review documents for this wage lawsuit, and Lawyer Art is assisting Lawyer Belle in connection with this case. Which of the following is correct?

Lawyer Art may review these documents if they are located in State A; Lawyer Art may review these documents if they are located in State B; Lawyer Art may review these documents if they are located in State C.

Which of the following business relationships is least problematic?

Lawyer C is representing a local gas station, and Lawyer C regularly purchases gas from this station at the going market rate.

Which of the following situations is most problematic?

Lawyer C publishes the following statement in a written ad: "I successfully obtain millions of dollars for my clients."

Which of the following lawyers is NOT subject to discipline?

Lawyer C, who is a litigator, engaged in a peremptory challenge of a juror solely because of the juror's gender.

Attorney Barbara is a solo practitioner representing the Bob, the buyer of a business, and Lawyer Sam is a solo practitioner representing Sara, the seller of a business. The sale of the business is supposed to be completed in one week, and the attorneys have been busy with finalizing the details of the transaction. Which of the following is correct?

Lawyer Chris, who is not working on the business deal and not currently representing anyone, believes he can provide a better legal representation to Bob, and so he sends a letter to Bob offering to represent him.

Lawyer Lauren is representing client Courtney in connection with the sale of Courtney's business. Which of the following is correct?

Lawyer Lauren may withdraw from the representation if the withdrawal can be accomplished without a material adverse effect on Courtney.

Lawyer Lera is an experienced divorce attorney. Wanda recently met with Lawyer Lera about the possibility of representing her in a divorce against her husband Harry, and Wanda revealed some sensitive financial and personal information about the marriage during the meeting. A few days after meeting with Wanda, and while Wanda was still deciding whether to hire Lawyer Lera, Wanda's husband Harry contacts Lawyer Lera and wishes to hire her as his attorney in the divorce. Which of the following is correct?

Lawyer Lera may represent Harry if both Wanda and Harry give informed consent, confirmed in writing.

Lawyer Libby is representing an individual plaintiff in a wage dispute with a company the plaintiff previously worked for, and the company is represented by counsel in this pending lawsuit. The chief executive officer (CEO) of the company calls Lawyer Libby to discuss the possibility of settlement. Which of the following best describes Lawyer Libby's ethical obligations?

Lawyer Libby may speak with the CEO if the company's lawyer consents.

Lawyer Libby represents client Carl in connection with a securities law dispute. Carl gives Lawyer Libby $20,000 as an advance to cover her hourly fees over the course of her representation, and Lawyer Libby places these funds in a trust account for clients. Lawyer Libby quickly completes the representation by settling the case with the opposing party. Lawyer Libby believes she earned $5,000 for the representation, based on her hourly rate, but Carl disputes the bill, claiming that he owes Libby nothing because she did almost no work and settled the case quickly. Which of the following best describes Lawyer Libby's duties regarding the $20,000 advance?

Lawyer Libby must promptly disburse $15,000 to Carl and hold the remaining $5,000 in the trust account until the fee dispute is resolved.

Pam, a non-lawyer physician, has been recommending her patients to hire Lawyer Lisa for their legal needs. Which of the following is correct?

Lawyer Lisa may give Pam a small fruit basket as an expression of appreciation; Lawyer Lisa may agree to refer clients to physician Pam, as long as the agreement is not exclusive and clients are informed of the existence and nature of this agreement; Lawyer Lisa may not pay Pam a referral fee for the recommendations.

Lawyer Logan formerly represented a local bank in connection with a data breach involving customer social security numbers from two years ago, and the representation of the bank is over. A customer of the bank wants to hire Lawyer Logan to sue the bank for the same data breach. Which of the following is correct?

Lawyer Logan may represent the customer if the bank gives informed consent, confirmed in writing.

Lawyer Logan formerly represented a local bank in connection with a data breach involving customer social security numbers from two years ago, and the representation of the bank is over. An employee of the bank wants to hire Lawyer Logan to sue the bank for a wage dispute. Which of the following is correct?

Lawyer Logan may represent the employee because the data breach and wage dispute are not substantially related matters.

Lawyer Lucian works as a solo practitioner, and he has an active bankruptcy practice and intellectual property practice. As he is getting older, Lawyer Lucian would like to reduce his work hours if possible, and he is considering to sell some or all of his practice. Which of the following is correct?

Lawyer Lucian may sell his entire practice to another solo practitioner as long as Lawyer Lucian ceases to engage in the private practice of law, Lawyer Lucian gives proper written notice to his clients, and existing fee agreements are honored; Lawyer Lucian may sell his entire bankruptcy practice, broken up in thirds, to three solo practitioners as long as Lawyer Lucian ceases to engage in bankruptcy practice; Lawyer Lucian gives proper written notice to his clients; and existing fee agreements are honored; Lawyer Lucian may sell his entire practice to another solo practitioner as long as Lawyer Lucian becomes an in-house counsel for a business; Lawyer Lucian gives proper written notice to his clients; and existing fee agreements are honored.

Lawyer Lucy is representing a client, Casey, in connection with a trial in a defamation lawsuit. Which of the following is correct?

Lawyer Lucy may ask Casey's brother to refrain from giving relevant information to the opposing party, as long as the brother's interests will not be adversely affected.

Lawyer Lucy specializes in criminal defense work. After watching the local news, she learns that the police department recently arrested Cole for assault, and Lawyer Lucy wants to defend Cole in the criminal proceeding against him. Which of the following is correct?

Lawyer Lucy may send Cole a letter offering to represent him for a reasonable fee.

Rule 8.4: Misconduct

Maintaining The Integrity Of The Profession It is professional misconduct for a lawyer to: (a) violate or attempt to violate the Rules of Professional Conduct, knowingly assist or induce another to do so, or do so through the acts of another; (b) commit a criminal act that reflects adversely on the lawyer's honesty, trustworthiness or fitness as a lawyer in other respects; (c) engage in conduct involving dishonesty, fraud, deceit or misrepresentation; (d) engage in conduct that is prejudicial to the administration of justice; (e) state or imply an ability to influence improperly a government agency or official or to achieve results by means that violate the Rules of Professional Conduct or other law; (f) knowingly assist a judge or judicial officer in conduct that is a violation of applicable rules of judicial conduct or other law; or (g) engage in conduct that the lawyer knows or reasonably should know is harassment or discrimination on the basis of race, sex, religion, national origin, ethnicity, disability, age, sexual orientation, gender identity, marital status or socioeconomic status in conduct related to the practice of law. This paragraph does not limit the ability of a lawyer to accept, decline or withdraw from a representation in accordance with Rule 1.16. This paragraph does not preclude legitimate advice or advocacy consistent with these Rules.

Rule 8.2: Judicial & Legal Officials

Maintaining The Integrity of The Profession (a) A lawyer shall not make a statement that the lawyer knows to be false or with reckless disregard as to its truth or falsity concerning the qualifications or integrity of a judge, adjudicatory officer or public legal officer, or of a candidate for election or appointment to judicial or legal office. (b) A lawyer who is a candidate for judicial office shall comply with the applicable provisions of the Code of Judicial Conduct.

Rule 8.3: Reporting Professional Misconduct

Maintaining The Integrity of The Profession (a) A lawyer who knows that another lawyer has committed a violation of the Rules of Professional Conduct that raises a substantial question as to that lawyer's honesty, trustworthiness or fitness as a lawyer in other respects, shall inform the appropriate professional authority. (b) A lawyer who knows that a judge has committed a violation of applicable rules of judicial conduct that raises a substantial question as to the judge's fitness for office shall inform the appropriate authority. (c) This Rule does not require disclosure of information otherwise protected by Rule 1.6 or information gained by a lawyer or judge while participating in an approved lawyers assistance program.

Rule 8.5: Disciplinary Authority; Choice of Law

Maintaining The Integrity of The Profession (a) Disciplinary Authority. A lawyer admitted to practice in this jurisdiction is subject to the disciplinary authority of this jurisdiction, regardless of where the lawyer's conduct occurs. A lawyer not admitted in this jurisdiction is also subject to the disciplinary authority of this jurisdiction if the lawyer provides or offers to provide any legal services in this jurisdiction. A lawyer may be subject to the disciplinary authority of both this jurisdiction and another jurisdiction for the same conduct. (b) Choice of Law. In any exercise of the disciplinary authority of this jurisdiction, the rules of professional conduct to be applied shall be as follows: (1) for conduct in connection with a matter pending before a tribunal, the rules of the jurisdiction in which the tribunal sits, unless the rules of the tribunal provide otherwise; an (2) for any other conduct, the rules of the jurisdiction in which the lawyer's conduct occurred, or, if the predominant effect of the conduct is in a different jurisdiction, the rules of that jurisdiction shall be applied to the conduct. A lawyer shall not be subject to discipline if the lawyer's conduct conforms to the rules of a jurisdiction in which the lawyer reasonably believes the predominant effect of the lawyer's conduct will occur.

Rule 8.1: Bar Admission & Disciplinary Matters

Maintaining The Integrity of The Profession An applicant for admission to the bar, or a lawyer in connection with a bar admission application or in connection with a disciplinary matter, shall not: (a) knowingly make a false statement of material fact; or (b) fail to disclose a fact necessary to correct a misapprehension known by the person to have arisen in the matter, or knowingly fail to respond to a lawful demand for information from an admissions or disciplinary authority, except that this rule does not require disclosure of information otherwise protected by Rule 1.6.

Attorney Art is defending Colby in a criminal proceeding accusing Colby of armed robbery. Attorney Art defends the proceeding in such a manner as to require that the prosecution prove every element of the robbery beyond a reasonable doubt. The case is still ongoing. Under which of the following situations is it mandatory for Attorney Art to withdraw from his representation of Colby?

Neither Attorney Art reasonably believes that Colby committed the armed robbery because an eyewitness believes he saw Colby, from a distance, engage in the robbery; nor Attorney Art knows that Colby committed the armed robbery because of indisputable video evidence from an eyewitness's cellphone conclusively demonstrates that Colby engaged in the robbery.

Lawyer Linda is the in-house counsel for Starbucks Coffee, a large, international coffee company. She is responsible for handling all the intellectual property matters, such as licensing, involving Starbucks. When consulting with a manager regarding an intellectual property matter, the manager informs Linda of illegal employment practices engaged in by Starbucks. Starbucks often requires its workers to stay after hours, off the clock, to close a store without getting properly paid, and this illegal employment practice can cause substantial financial liability to Starbucks. Which of the following is correct?

Neither Lawyer Linda must inform the board of directors of Starbucks about these violations of employment law, nor Lawyer Linda must inform a government agency in charge of labor and employment issues about these violations of employment law.

During a negotiation for a settlement in a personal injury case, Attorney Alex tells the opposing attorney that his client will not accept less than $25,000 to settle the case. However, Attorney Alex knows that his client will gladly accept anything over $5,000 to settle. Is Attorney Alex subject to discipline for falsely stating that his client will not accept less than $25,000?

No, because this statement is acceptable in negotiations.

Attorney Art is a personal injury lawyer, and using public court records, he sees that a negligence lawsuit was recently filed against David, a driver involved in a car accident. Attorney Art visits David in person and offers to represent him in the negligence lawsuit. Is Attorney Art subject to discipline?

No, if Attorney Art is offering to provide the legal services pro bono; No, if David is a prior client; No, if David is a close personal friend.

Attorney Abby reads in the newspaper that Carl was just arrested for burglary. Hoping to earn a significant legal fee, Attorney Abby reaches out to Carl to ask him to hire her. Is Attorney Abby subject to discipline?

No, if she contacts Carl via email; No, if she contacts Carl via regular mail.

During a criminal trial for burglary, Attorney Abby is representing the defendant, and Attorney Abby hears the prosecutor mistakenly tell the judge that Abby's client has no prior criminal record. But in fact, Attorney Abby's client does have a past criminal record, which will significantly impact the sentencing in this case. The case is ongoing. Which of the following best describes Attorney Abby's ethical duties?

None of the following: Attorney Abby must take remedial measures, including, if necessary, disclosure of her client's prior record to the court; Attorney Abby must immediately inform the court of her client's prior record; Attorney Abby must immediately inform the prosecution of her client's prior record.

In which of the following circumstances is a lawyer required to reveal information normally protected by the duty of confidentiality?

None of the following: The lawyer reasonably believes the client is about to engage in wrongdoing that will result in substantial bodily harm to others; The lawyer knows that the client is about to engage in wrongdoing that will result in substantial bodily harm to others; The lawyer knows that the client is about to engage in wrongdoing that will result in substantial financial injury to others.

Rule 6.3: Membership in Legal Services Organization

Public Service A lawyer may serve as a director, officer or member of a legal services organization, apart from the law firm in which the lawyer practices, notwithstanding that the organization serves persons having interests adverse to a client of the lawyer. The lawyer shall not knowingly participate in a decision or action of the organization: (a) if participating in the decision or action would be incompatible with the lawyer's obligations to a client under Rule 1.7; or (b) where the decision or action could have a material adverse effect on the representation of a client of the organization whose interests are adverse to a client of the lawyer.

Rule 6.2: Accepting Appointments

Public Service A lawyer shall not seek to avoid appointment by a tribunal to represent a person except for good cause, such as: (a) representing the client is likely to result in violation of the Rules of Professional Conduct or other law; (b) representing the client is likely to result in an unreasonable financial burden on the lawyer; or (c) the client or the cause is so repugnant to the lawyer as to be likely to impair the client-lawyer relationship or the lawyer's ability to represent the client.

Rule 6.1: Voluntary Pro Bono Publico Service

Public Service Every lawyer has a professional responsibility to provide legal services to those unable to pay. A lawyer should aspire to render at least (50) hours of pro bono publico legal services per year. In fulfilling this responsibility, the lawyer should: (a) provide a substantial majority of the (50) hours of legal services without fee or expectation of fee to: Looked at as being more important than section (b), majority = 30-35 hours (1) persons of limited means or (2) charitable, religious, civic, community, governmental and educational organizations in matters that are designed primarily to address the needs of persons of limited means; and (b) provide any additional services through: (1) delivery of legal services at no fee or substantially reduced fee to individuals, groups or organizations seeking to secure or protect civil rights, civil liberties or public rights, or charitable, religious, civic, community, governmental and educational organizations in matters in furtherance of their organizational purposes, where the payment of standard legal fees would significantly deplete the organization's economic resources or would be otherwise inappropriate; not under the poverty line (2) delivery of legal services at a substantially reduced fee to persons of limited means; or (3) participation in activities for improving the law, the legal system or the legal profession. In addition, a lawyer should voluntarily contribute financial support to organizations that provide legal services to persons of limited means.

Client Chris suggests to his attorney Albert that they should submit fraudulent responses to the opposing party in connection with recent discovery requests in a civil lawsuit. Which of the following is correct?

Submitting fraudulent responses in discovery is unethical.; Attorney Albert has a duty to inform Chris of his ethical limitations because Chris is expecting unethical assistance with discovery.

Lawyer Lance is admitted to practice law only in State A. He is about to file a negligence lawsuit in court in State B against a company located in State B. In order to prepare for the upcoming lawsuit and gather evidence for the case, Lawyer Lance interviews some potential witnesses located in State C in preparation for the future lawsuit to be filed in State B, and these witness statements are relevant to the future lawsuit. During the course of these interviews in State C, some of which are extensive, Lance makes certain statements to these witnesses, and these statements could be problematic under the ethics rules of some jurisdictions. Which of the following is correct?

The ethics rules of State A will govern any disciplinary proceedings involving these statements if the predominant effect of Lawyer Lance's statements is in State A.

Lawyer Lance is admitted to practice law only in State A. He has already filed a negligence lawsuit in court in State B against a company located in State B. After the lawsuit is filed, in connection with the discovery phase of the litigation, Lawyer Lance collects some evidence in State C, and this evidence is relevant to the pending lawsuit. During the course of the collection of evidence in State C, Lance makes certain payments to potential witnesses in order to gather the evidence, and these payments made by Lance could be problematic under the ethics rules of some jurisdictions. Which of the following is correct?

The ethics rules of State B will govern any disciplinary proceedings involving these payments because the lawsuit is pending in State B.

Darryl is arrested for armed robbery. The prosecution discovers someone named Walter who witnessed the robbery and who identified someone other than Darryl as the robber. Lawyer Leo is representing Darryl and is unaware of Walter. Which of the following is correct?

The prosecution must reveal Walter to Lawyer Leo.

Judge Jackie served as the trial judge in the case of Phyllis v. Dylan, which is still ongoing. She joins the ABC Law Firm, a very popular firm in hypo city, which represents Dylan in this ongoing case. Which of the following is correct?

The other lawyers in the ABC Law Firm may continue representing Dylan if Judge Jackie is screened from the matter, given no part of the fee, and written notice is given to all the parties and the court.

Rule 4.3: Dealing with Unrepresented Person

Transactions With Persons Other Than Clients In dealing on behalf of a client with a person who is not represented by counsel, a lawyer shall not state or imply that the lawyer is disinterested. When the lawyer knows or reasonably should know that the unrepresented person misunderstands the lawyer's role in the matter, the lawyer shall make reasonable efforts to correct the misunderstanding. The lawyer shall not give legal advice to an unrepresented person, other than the advice to secure counsel, if the lawyer knows or reasonably should know that the interests of such a person are or have a reasonable possibility of being in conflict with the interests of the client.

Rule 4.2: Communication with Person Represented by Counsel

Transactions With Persons Other Than Clients In representing a client, a lawyer shall not communicate about the subject of the representation with a person the lawyer knows to be represented by another lawyer in the matter, unless the lawyer has the consent of the other lawyer or is authorized to do so by law or a court order.

Rule 4.1: Truthfulness in Statements to Others

Transactions With Persons Other Than Clients In the course of representing a client a lawyer shall not knowingly: (a) make a false statement of material fact or law to a third person; or (b) fail to disclose a material fact to a third person when disclosure is necessary to avoid assisting a criminal or fraudulent act by a client, unless disclosure is prohibited by Rule 1.6.

Lawyer Linda is representing client Cooper in connection with a complex debt restructuring deal. Because the representation involves cutting-edge legal issues, Lawyer Linda wants to ask Cooper to sign a waiver that would limit any liability for legal malpractice to $5,000. May Lawyer Linda do so?

Yes, as long as Cooper is represented by an independent lawyer regarding the signing of the waiver.

Driver Dolan hired Attorney Alissa to represent him in a criminal prosecution alleging that Dolan was driving while intoxicated in a jurisdiction where there is an increased penalty for a second offense of drunk driving. Dolan informs Attorney Alissa that his driver's license had been obtained under a fake name because his prior license had already been suspended for driving while intoxicated, and he confesses to Attorney Alissa that his real name is Dudley. He asks Attorney Alissa not to disclose his true name during the course of the representation, and he informs Attorney Alissa that, if called as a witness, he would definitely give his fake name to avoid the increased penalty for a second offense. Attorney Alissa tells him that, in order properly to defend the case, she must call him as a witness. During the trial, Attorney Alissa calls Dudley as a witness and, in response to Attorney Alissa's question "What is your name?," Dudley gives his fake name and not his true name. Is Attorney Alissa subject to discipline?

Yes, because Attorney Alissa knowingly elicited false testimony.

Attorney Andy entered into a written retainer agreement signed by client Caleb, who is the defendant in a criminal case. Caleb agreed in writing to transfer title to Caleb's car to Attorney Andy if Attorney Andy obtained a jury verdict declaring Caleb to be innocent. The jury eventually declares that Caleb is innocent. Is Attorney Andy subject to discipline for entering into this agreement?

Yes, because Attorney Andy agreed to a contingent fee in a criminal case.

Attorney Art is currently representing Acme Corp. in negotiating a copyright license with Bristol, Inc., which is represented by another attorney. Pleased with the professionalism of Attorney Art, Acme would like to hire Attorney Art in connection with an unrelated breach of contract lawsuit against Caddy Corp. May Attorney Art represent Acme in the lawsuit?

Yes, because there is no improper conflict of interest.

Carl wants to hire Attorney Annie, a top employment lawyer in the city, to negotiate and draft an employment contract on his behalf to become the next manager of Roy's Restaurant. Unbeknownst to Carl, Attorney Annie's husband owns half of Roy's Restaurant. May Attorney Annie represent Carl?

Yes, if Carl gives informed consent, confirmed in writing.

Lawyer Lucy is currently defending a local supermarket in a slip-and-fall lawsuit filed by a customer Chris. Chris would like to hire Lawyer Lucy in connection with an unrelated real estate matter in purchasing some land from Daniel. May Lawyer Lucy represent Chris in the unrelated real estate matter?

Yes, if Chris and the supermarket give informed consent, confirmed in writing.

Lawyer Libby is admitted to practice law only in State A. A client has hired Libby to travel to State B to give advice about federal environmental law in connection with the construction of a factory in State B. May Lawyer Libby provide these legal services in State B?

Yes, if Lawyer Libby is an expert on federal environmental law.

Attorney Art is currently representing Acme Corp. in negotiating a copyright license with Bristol, Inc., which is represented by another attorney. Pleased with the professionalism of Attorney Art, Bristol, Inc. would like to hire Attorney Art in connection with an unrelated breach of contract lawsuit against Caddy Corp. May Attorney Art represent Bristol in the lawsuit?

Yes, if both Acme and Bristol give informed consent, confirmed in writing.

Christine is suing a local car dealership, Carolina Ford, as well as its manager, Mark, as co-defendants in a negligence lawsuit because the dealership's service department appears to have been negligent in repairing Christine's car. Both the car dealership and its manager would like to hire the same attorney, Attorney Agatha, to defend them, and as a defense, their core arguments would be that negligence did not occur and that Christine was herself contributorily negligent in how she operated and maintained her own car. May Attorney Agatha represent both the car dealership and the manager as co-defendants?

Yes, if the car dealership and manager give informed consent, confirmed in writing.


Kaugnay na mga set ng pag-aaral

STATS 2381 Ch. 2,3,4,6,7,8 All True/False Quizzes

View Set

Trigonometry DAT quantitative reasoning

View Set

Corporate Finance Test 2 - Ch. 5, 6, 7

View Set

Abraham Maslow: Theory of Human Motivation

View Set

CompTIA CertMaster Linux+ LXO-103 ALL

View Set

ECO 101 (Module 10 - Monopolistic Competition and Oligopoly)

View Set

SPRING AFROTC AS100 Final Study Guide

View Set